Board logo

標題: 99基隆女中 [打印本頁]

作者: johncai    時間: 2010-8-10 17:36     標題: 99基隆女中

f(x)=x^3-x^2-2x+2
(1)求過(0,1)的切線
(2)求切線與f(x)圍成區域面積

謝謝!
(題目X^2係數改成-1,PTT有人幫忙解了)
作者: weiye    時間: 2010-8-11 19:06

請電腦幫我算(依循下列流程自己手算也OK),

作者: 八神庵    時間: 2010-8-19 23:47     標題: 99基隆女中

今日公佈題目
雖然今年正式戰役已經結束
但是仍舊提供考題

附件: 99基隆女中.zip (2022-6-10 09:01, 375.75 KB) / 該附件被下載次數 9468
https://math.pro/db/attachment.php?aid=292&k=88d167ff2d8d3a45ebfe1b02104b4275&t=1714605059
作者: ivan_jaw    時間: 2010-8-20 11:42

ppt 的其中一個是本人
我再轉PO過來

\( f'(x)=3x^2-2x-2 \)

令切點 \( (a,f(a)) \)  , \(a\) 為實數

所以切線為  \( y=(3a^2-2a-2)x+1 \)

則  \( (3a^2-2a-2)x+1=x^3-x^2-2x+2 \)必有\(a\)的根

=> \(x=a\) 代入 可得\( 2a^3-a^2-1=0\)

=>\( (a-1)(2a^2+a+1)=0\)

=>\( a=1\)


故切線為 \( y=-x+1\)

切點為\( (1,0)\)

則另一交點為\( (-1,f(-1))\)
   
所以\(\displaystyle \int_{1}^{-1}(f(x)-(-x+1))dx =\frac{4}{3}\)
作者: bugmens    時間: 2010-8-20 21:07

4.設\( \theta \)為銳角,則\( 64sec^2 \theta+csc^2 \theta+16sec \theta csc \theta \)的最小值為?
[提示]
利用\( \displaystyle \frac{8}{cos \theta}+\frac{1}{sin \theta} \)作廣義的柯西不等式

我的教甄準備之路 廣義的柯西不等式
https://math.pro/db/viewthread.php?tid=661&page=1#pid1075

我將題目重新打字,要用Open Office才能開啟

附件: 99基隆女中LibreOffice檔.rar (2014-10-19 17:33, 99.66 KB) / 該附件被下載次數 8406
https://math.pro/db/attachment.php?aid=293&k=2744297b54e078f2177d62f1b26cc462&t=1714605059
作者: mathyen    時間: 2010-8-24 11:55

引用:
原帖由 bugmens 於 2010-8-20 09:07 PM 發表
4.設\( \theta \)為銳角,則\( 64sec^2 \theta+csc^2 \theta+16sec \theta csc \theta \)的最小值為?
[提示]
利用\( \displaystyle \frac{8}{cos \theta}+\frac{1}{sin \theta} \)作廣義的柯西不等式

我的教甄準備之路 ...
我是利用微積分求得tan 在代回去算的
作者: kittyyaya    時間: 2010-10-28 02:37

想請問填充第 2 題和計算第 4 題,謝謝
作者: weiye    時間: 2010-10-28 11:57

填充第 2 題:已知兩圓相交於 \(A, B\) 兩點,其中一圓上另有 \(C,D,E,F\) 四點,第二圓上另有 \(G, H, I, J\) 四點。若此十點中,只有 \(B, C, G\) 三點共線,除此之外,再沒有任意三點共線,則此十點共可決定_______個相異圓。


解答:

\(C^{10_3}-C^6_3-C^6_3-C^3_3+2=81.\)

解釋:

十點任取三點是最多的圓的情況,
扣掉 ABCDEF 六點中任取三點的情況,
扣掉 ABGHUJ 六點中任取三點的情況,
扣掉 BCG 三點中任取三點的情況,
加回來 ABCDEF 六點恰決定的〝一個〞圓,
加回來 ABGHIJ 六點恰決定的〝一個〞圓。


計算第四題:如右圖,邊長為一單位的正方體平放在於桌上,在其上方放置若干個小正方體堆成塔形。已知每一個上面正方體之下底面的四個頂點是下面相鄰正方體之上底面的各邊中點,若所有正方體暴露在外面部分的面積和超過8.9999,則正方體的個數至少須多少個。

※ 在上方 bugmens 回覆的檔案中有寫此題來自建中通訊解題第15期,找建中通訊解題就知道做法了。
作者: icesnow1129    時間: 2011-5-10 15:12

引用:
原帖由 bugmens 於 2010-8-20 09:07 PM 發表
4.設\( \theta \)為銳角,則\( 64sec^2 \theta+csc^2 \theta+16sec \theta csc \theta \)的最小值為?
[提示]
利用\( \displaystyle \frac{8}{cos \theta}+\frac{1}{sin \theta} \)作廣義的柯西不等式

我的教甄準備之路 ...
想請教如何利用廣義柯西求解?
有了提示仍然不知所以然....
感謝!!
作者: dream10    時間: 2011-5-10 21:12

原式即求\( \displaystyle \left( \frac{8}{cos \theta}+\frac{1}{sin \theta} \right)^2 \)
根據廣義柯西
\( \displaystyle \left[ \left( \root 3 \of {\frac{8}{cos \theta}} \right)^3+\left( \root 3 \of {\frac{1}{sin \theta}} \right)^3 \right]
\left[ \left( \root 3 \of {\frac{8}{cos \theta}} \right)^3+\left( \root 3 \of {\frac{1}{sin \theta}} \right)^3 \right]
\left[ \left( \root 3 \of {cos^2 \theta} \right)^3+\left( \root 3 \of {sin^2 \theta} \right)^3 \right] \ge \)
\( \displaystyle \left( \root 3 \of {\frac{8}{cos \theta} \times \frac{8}{cos \theta}\times cos^2 \theta}+
\root 3 \of {\frac{1}{sin \theta}\times \frac{1}{sin \theta} \times sin^2 \theta} \right)^3=5^3=125 \)
作者: 阿光    時間: 2011-11-28 20:31

想請教計算題第2題 謝謝
作者: weiye    時間: 2011-11-28 21:18     標題: 回復 11# 阿光 的帖子

計算題第 2 題
已知數列\(\langle\;a_n\rangle\;\)滿足\(2a_{n+1}+a_n=3\),且\(a_1=10\),設前\(n\)項和為\(S_n\),則滿足不等式\(\displaystyle |\;S_n-n-6|\;<\frac{1}{250}\)的最小正整數\(n\)為何?
[解答]
\(\displaystyle 2 a_n+a_{n-1}=3\Rightarrow (a_n -1) = -\frac{1}{2}(a_{n-1} -1)\)

開始條列~

\(\displaystyle (a_n -1) = -\frac{1}{2}(a_{n-1} -1)\)

\(\displaystyle (a_{n-1} -1) = -\frac{1}{2}(a_{n-2} -1)\)

\(\displaystyle (a_{n-2} -1) = -\frac{1}{2}(a_{n-3} -1)\)

    \(\cdots\)

\(\displaystyle (a_2 -1) = -\frac{1}{2}(a_1 -1)\)

將上列各式全部乘起來,

可得 \(\displaystyle a_n - 1 = (-\frac{1}{2})^{n-1}(a_1-1) = 9\times(\frac{-1}{2})^{n-1}\)

\(\displaystyle \Rightarrow a_n = 1+ 9\times(\frac{-1}{2})^{n-1}\)

然後,

\(\displaystyle S_n = n\times1+\frac{9(1-(\frac{-1}{2})^n)}{1-\frac{-1}{2}}\)

  \(\displaystyle =n+6(1-(\frac{-1}{2})^n)\)

因此,

\(\displaystyle \left|S_n-n-6\right| = \left|6(\frac{-1}{2})^n\right|=\frac{3}{2^{n-1}}<\frac{1}{250}\)

\(\displaystyle \Rightarrow 2^{n-1}>250\times3=750\)

因為 \(2^9=512, 2^{10}=1024\),所以 \(n-1\) 至少為 \(10\),

故,\(n\) 至少為 \(11.\)
作者: meifang    時間: 2012-7-4 00:55

我想要問填充第9題和第12題
第9題我有兩個想法 1.幾何圖形解 但是不知所措 2.柯西不等式 但是好像多一個式子 等號不成立
第12題 我將60度拆成兩個角度 再用餘弦定理 但是式子列出來 我就不知道怎麼辦了
作者: katama5667    時間: 2012-7-4 14:00     標題: 回復 13# meifang 的帖子

填充9
設\(P(4,3,1)\),\(Q\)為圓\(\cases{x^2+(y-1)^2+(z-5)^2=13 \cr x+2y+2z=3}\)上之動點,求\(\overline{PQ}\)之最小值   
[解答]
這題所求發生在 \(P,O,G,O',P'\)皆在同一平面時,

其中 \(O',P'\) 為 \(O,P\) 在平面 \(E:x+2y+2z=3\) 的垂足

其圖如下:

令 \(O'(t,1+2t,5+2t),P'(4+s,3+2s,1+2s) \),代入平面 \(E\) 中,求得 \(t=s=-1\)  

則 \(O'(-1,-1,3),P' (3,1,-1) \) ,求出  \(\overline{OO'}=\overline{PP'}=3,\overline{O'P'}=6,\overline{O'G}=2\)

所以 \(\overline{GP'}=4\Rightarrow \overline{GP}=5\)

填充12題

將\(\overline{PQ}\)連起來,交 \(\overline{ST}\) 在 \(R\)

因為 \(\Delta PSR \) 與 \(\Delta QTR\) 相似,若令 \(\overline{RS}=x\) ,則  \(\overline{RT}=2x\)

再由 \(\angle PAB=\angle QAB=30^{\circ}\) ,可知 \(\overline{AS}=\sqrt{3}\) 且 \(\overline{AT}=2\sqrt{3}\)

所以 \(\overline{AT}=\overline{AS}+x+2x\Rightarrow x=\frac{1}{\sqrt{3}}\)

再推得 \(\overline{PR}=2x, ~\overline{RQ}=2x\)

故 \(\overline{PQ}=6x=\frac{6}{\sqrt{3}}=2\sqrt{3}\)

圖片附件: image.png (2014-10-19 17:09, 15.32 KB) / 該附件被下載次數 4927
https://math.pro/db/attachment.php?aid=2561&k=74b614b52b5f3e20a1751a3d22195d43&t=1714605059


作者: meifang    時間: 2012-7-5 12:11     標題: 回復 14# katama5667 的帖子

謝謝樓上老師 剛剛才發現第12題 我沒看到\(\bar{AB}\)平分那個角
作者: mathelimit    時間: 2014-10-19 14:54

請教填充第一題的想法~ QAQ
作者: thepiano    時間: 2014-10-19 15:52     標題: 回復 16# mathelimit 的帖子

填充第 1 題
設\(\Delta ABC\)中,已知\(\overline{BC}\)與\(y\)軸垂直,若\(A(2,9)\),內切圓圓心為\((1,1)\),半徑為4,則\(\Delta ABC\)的垂心\(H\)坐標為   
[解答]
易知直線 BC 之方程式為 y = -3
利用 (1,1) 到直線  y - 9 = m(x - 2) 的距離為 4,可求出直線 AB 和直線 AC 之方程式
剩下就簡單了
作者: mathelimit    時間: 2014-10-19 17:00     標題: 回復 17# thepiano 的帖子

解出來了,謝謝。^^b
一開始感覺很麻煩,就不敢下手了...。
作者: anyway13    時間: 2021-9-4 12:55     標題: 請教第7題

請問板上老師  第七題   要怎麼拆解呢

算到阿發+beta=323度   tan((阿發+beta)/2)==-1/3  tan((beta-45度)/2)

但是一直作不出sin阿發*cos阿發=???
作者: thepiano    時間: 2021-9-4 22:01     標題: 回復 19# anyway13 的帖子

第7題
已知\(\displaystyle \frac{3}{4}\pi<\alpha<\pi\),\(\displaystyle \frac{3}{4}\pi<\beta<\pi\),且\(\displaystyle sin(\alpha+\beta)=-\frac{3}{5}\),\(\displaystyle sin(\beta-\frac{\pi}{4})=\frac{12}{13}\),則\(\displaystyle sin(\alpha+\frac{\pi}{4})=\)   
[提示]
\(\sin \left( \alpha +\frac{\pi }{4} \right)=\sin \left[ \left( \alpha +\beta  \right)-\left( \beta -\frac{\pi }{4} \right) \right]\)
作者: anyway13    時間: 2021-9-4 23:16     標題: 回復 21# thepiano 的帖子

原來是這樣阿    謝謝鋼琴老師指導




歡迎光臨 Math Pro 數學補給站 (https://math.pro/db/) 論壇程式使用 Discuz! 6.1.0